首页 > 电气工程及其自动化> 电气安全
题目内容 (请给出正确答案)
[主观题]

已知系统H(z)的频率响应如图A-14所示,系统H1(z)定义为H1(z)=H(z3),试画出系统H1(z)的频率响应H1(ejΩ)。

已知系统H(z)的频率响应如图A-14所示,系统H1(z)定义为H1(z)=H(z3),试画出系统H1(z)的频率响应H1(e)。

查看答案
答案
收藏
如果结果不匹配,请 联系老师 获取答案
您可能会需要:
您的账号:,可能还需要:
您的账号:
发送账号密码至手机
发送
安装优题宝APP,拍照搜题省时又省心!
更多“已知系统H(z)的频率响应如图A-14所示,系统H1(z)定…”相关的问题
第1题
已知一离散系统Hd(z)的频率响应Hd(ejΩ),如图A-22所示,系统H(z)定义为H(z)=Hd(-z)。

已知一离散系统Hd(z)的频率响应Hd(e),如图A-22所示,系统H(z)定义为H(z)=Hd(-z)。

点击查看答案
第2题
已知一理想系统的频率响应H(ejΩ)如图A-18所示,试求出周期序列通过该系统的响应。

已知一理想系统的频率响应H(e)如图A-18所示,试求出周期序列通过该系统的响应

点击查看答案
第3题
如图1-2所示,h(n)是方框所包含的LTI系统的单位脉冲响应,h(n)的Z变换H(z)在0<rmin<|z|<rmax<∞存在。

如图1-2所示,h(n)是方框所包含的LTI系统的单位脉冲响应,h(n)的Z变换H(z)在0<rmin<|z|<rmax<∞存在。

点击查看答案
第4题
考虑如图2-17所示的系统,子系统S1和S2的频率响应H1(ejω)和H2(ejω)满足H1(ejω)=0,当|ω|≤0.2π;H2(ejω)=0,当0.

考虑如图2-17所示的系统,子系统S1和S2的频率响应H1(e)和H2(e)满足H1(e)=0,当|ω|≤0.2π;H2(e)=0,当0.4π<|ω|≤π。又已知输入x(n)带限到0.3π,即X(e)=0,当0.3π<|ω|≤π。请问y(n)的离散时间傅里叶变换(DTFT)Y(e)在-π≤ω<π的什么范围内为零?

点击查看答案
第5题
已知一个电压串联负反馈放大器的开环频率特性如图2.5.5所示。是写出它的频率响应表达式,并判断当
采用kfv=0.01的反馈系数构成闭环系统时能否稳定工作?若不能稳定工作,求能够稳定工作的最大反馈系数为多少?

点击查看答案
第6题
IIR滤波技术的一个应用就是产生和恢复用于按键电话机的双音多频信号(DTMF)。按键电话机的键盘如图3-12所示。

IIR滤波技术的一个应用就是产生和恢复用于按键电话机的双音多频信号(DTMF)。按键电话机的键盘如图3-12所示。

每当按下一个按键时,产生一对音频信号,其中一个信号对按键的行编码,另外一个信号对列编码。构成双音多频信号对的单音可以通过对IIR滤波器上加一个脉冲函数来产生。滤波器输出的z变换是传输函数H(z)与输入的Z变换X(z)的乘积

Y(z)=H(z)X(z)

脉冲函数的Z变换为X(z)=1,所以此时Y(z)=H(z),Y(Ω)=H(Ω)。这样,输出信号的频谱与滤波器的频率响应是一样的。单音由正弦波信号产生,所以能够产生单音信号的滤波器传输函数与正弦波信号的Z变换相同

Ω0为待求音的数字频率。该音频发生器的差分方程为

y(n)=2cosΩ0y(n-1)-y(n-2)+sinΩ0x(n-1)

令a1=-2cosΩ0,a2=1,b1=sinΩ0。如图3-13所示,为了直观,采用直接Ⅰ型表示。产生一个DTMF信号需要两个这种音频发生滤波器。下面的例子说明了如何设计音频发生器来产生指定的音频信号。

要设计滤波器产生的单音信号为1209Hz,采样频率为8kHz,首先画出滤波器形状,然后求出滤波器的脉冲响应,最后画出脉冲响应的幅度频谱。

点击查看答案
第7题
采样系统如图7-5所示,其中G(s)对应的z变换式为G(z),已知:,(K>0),问:闭环系统稳定时,K应如何取值?

采样系统如图7-5所示,其中G(s)对应的z变换式为G(z),已知:,(K>0),问:闭环系统稳定时,K应如何取值?

点击查看答案
第8题
采样系统如图7-12所示,其中G(s)对应的Z变换式为G(z),已知: ,(K>0) 问:闭环系统稳定时,K应如何取值?

采样系统如图7-12所示,其中G(s)对应的Z变换式为G(z),已知:

,(K>0)

问:闭环系统稳定时,K应如何取值?

点击查看答案
第9题
某离散系统的系统函数为 且H(z)被分解为如图7—12所示的级联形式。 求H2(z)的表达式,并画出直

某离散系统的系统函数为

且H(z)被分解为如图7—12所示的级联形式。

求H2(z)的表达式,并画出直接Ⅱ型的框图或流图。

点击查看答案
第10题
已知离散系统结构如图7-14所示。 其中:x(k)=x(k)-x(k-1),c(k)=1.5c(k-1)-5c(k-2)+4x(k-1)。

已知离散系统结构如图7-14所示。

其中:x(k)=x(k)-x(k-1),c(k)=1.5c(k-1)-5c(k-2)+4x(k-1)。试确定系统的脉冲传递函数G1(z)和G2(z)并判断系统的稳定性。

点击查看答案
第11题
已知系统函数 试给出实现H(z)的框图及数据处理算法。

已知系统函数

试给出实现H(z)的框图及数据处理算法。

点击查看答案
退出 登录/注册
发送账号至手机
密码将被重置
获取验证码
发送
温馨提示
该问题答案仅针对搜题卡用户开放,请点击购买搜题卡。
马上购买搜题卡
我已购买搜题卡, 登录账号 继续查看答案
重置密码
确认修改